Does there exist positive rational $s$ for which $zeta(s)$ is a positive integer? Announcing the arrival of Valued Associate #679: Cesar Manara Planned maintenance scheduled April 17/18, 2019 at 00:00UTC (8:00pm US/Eastern)Riemann zeta function at odd positive integersAre the amplitudes of these frequency spikes equal to 1 when the real part of the complex number “s” is equal to one half?For which positive integers n does there exist a prime whose digits sum to n?Determine one triple of positive integerProbability number is divisible by half the square of a prime?Infinitely many solutions to divisor equationDoes there exist $a in mathbbQ$ such that $a^2 - a + 1$ is a square?Irrationality of the values of the prime zeta functionA new proof that $zeta(2n+1)$ for integer $n>1$ is irrational?Question related to derived formula for zeta-zero counting function

How discoverable are IPv6 addresses and AAAA names by potential attackers?

Are my PIs rude or am I just being too sensitive?

Can Pao de Queijo, and similar foods, be kosher for Passover?

How do I stop a creek from eroding my steep embankment?

When to stop saving and start investing?

What does the "x" in "x86" represent?

What do you call a plan that's an alternative plan in case your initial plan fails?

The logistics of corpse disposal

Do I really need recursive chmod to restrict access to a folder?

Does accepting a pardon have any bearing on trying that person for the same crime in a sovereign jurisdiction?

I am not a queen, who am I?

Why is "Captain Marvel" translated as male in Portugal?

Single word antonym of "flightless"

What is this single-engine low-wing propeller plane?

What is the musical term for a note that continously plays through a melody?

If a contract sometimes uses the wrong name, is it still valid?

Withdrew £2800, but only £2000 shows as withdrawn on online banking; what are my obligations?

What are the motives behind Cersei's orders given to Bronn?

How does cp -a work

Should gear shift center itself while in neutral?

Why are there no cargo aircraft with "flying wing" design?

What LEGO pieces have "real-world" functionality?

How to motivate offshore teams and trust them to deliver?

If Jon Snow became King of the Seven Kingdoms what would his regnal number be?



Does there exist positive rational $s$ for which $zeta(s)$ is a positive integer?



Announcing the arrival of Valued Associate #679: Cesar Manara
Planned maintenance scheduled April 17/18, 2019 at 00:00UTC (8:00pm US/Eastern)Riemann zeta function at odd positive integersAre the amplitudes of these frequency spikes equal to 1 when the real part of the complex number “s” is equal to one half?For which positive integers n does there exist a prime whose digits sum to n?Determine one triple of positive integerProbability number is divisible by half the square of a prime?Infinitely many solutions to divisor equationDoes there exist $a in mathbbQ$ such that $a^2 - a + 1$ is a square?Irrationality of the values of the prime zeta functionA new proof that $zeta(2n+1)$ for integer $n>1$ is irrational?Question related to derived formula for zeta-zero counting function










25












$begingroup$



Does there exist positive rational $s$ for which the Riemann Zeta function $zeta(s) in N$ or equivalently, does there exist finite positive integers $ell,m$ and $n$ such that $$zetaleft(1+dfracellmright) = n$$




My progress: Using the method described in my answer below, I am running a computer program, I have been able to show that if there is a solution then $l > 2.21times 10^4$.










share|cite|improve this question











$endgroup$







  • 12




    $begingroup$
    I would expect that $zeta(r)$ is irrational for all rationals $r > 1$, but to prove this may be beyond the current state of the art. We don't even know that $zeta(5)$ is irrational.
    $endgroup$
    – Robert Israel
    Aug 19 '18 at 5:54






  • 1




    $begingroup$
    If $zeta$ denotes the Riemann zeta function, please include it in your question.
    $endgroup$
    – Klangen
    Aug 22 '18 at 13:07















25












$begingroup$



Does there exist positive rational $s$ for which the Riemann Zeta function $zeta(s) in N$ or equivalently, does there exist finite positive integers $ell,m$ and $n$ such that $$zetaleft(1+dfracellmright) = n$$




My progress: Using the method described in my answer below, I am running a computer program, I have been able to show that if there is a solution then $l > 2.21times 10^4$.










share|cite|improve this question











$endgroup$







  • 12




    $begingroup$
    I would expect that $zeta(r)$ is irrational for all rationals $r > 1$, but to prove this may be beyond the current state of the art. We don't even know that $zeta(5)$ is irrational.
    $endgroup$
    – Robert Israel
    Aug 19 '18 at 5:54






  • 1




    $begingroup$
    If $zeta$ denotes the Riemann zeta function, please include it in your question.
    $endgroup$
    – Klangen
    Aug 22 '18 at 13:07













25












25








25


8



$begingroup$



Does there exist positive rational $s$ for which the Riemann Zeta function $zeta(s) in N$ or equivalently, does there exist finite positive integers $ell,m$ and $n$ such that $$zetaleft(1+dfracellmright) = n$$




My progress: Using the method described in my answer below, I am running a computer program, I have been able to show that if there is a solution then $l > 2.21times 10^4$.










share|cite|improve this question











$endgroup$





Does there exist positive rational $s$ for which the Riemann Zeta function $zeta(s) in N$ or equivalently, does there exist finite positive integers $ell,m$ and $n$ such that $$zetaleft(1+dfracellmright) = n$$




My progress: Using the method described in my answer below, I am running a computer program, I have been able to show that if there is a solution then $l > 2.21times 10^4$.







real-analysis number-theory prime-numbers analytic-number-theory riemann-zeta






share|cite|improve this question















share|cite|improve this question













share|cite|improve this question




share|cite|improve this question








edited Mar 26 at 6:30









J. M. is a poor mathematician

61.3k5152291




61.3k5152291










asked Aug 19 '18 at 5:35









Nilotpal Kanti SinhaNilotpal Kanti Sinha

4,67121641




4,67121641







  • 12




    $begingroup$
    I would expect that $zeta(r)$ is irrational for all rationals $r > 1$, but to prove this may be beyond the current state of the art. We don't even know that $zeta(5)$ is irrational.
    $endgroup$
    – Robert Israel
    Aug 19 '18 at 5:54






  • 1




    $begingroup$
    If $zeta$ denotes the Riemann zeta function, please include it in your question.
    $endgroup$
    – Klangen
    Aug 22 '18 at 13:07












  • 12




    $begingroup$
    I would expect that $zeta(r)$ is irrational for all rationals $r > 1$, but to prove this may be beyond the current state of the art. We don't even know that $zeta(5)$ is irrational.
    $endgroup$
    – Robert Israel
    Aug 19 '18 at 5:54






  • 1




    $begingroup$
    If $zeta$ denotes the Riemann zeta function, please include it in your question.
    $endgroup$
    – Klangen
    Aug 22 '18 at 13:07







12




12




$begingroup$
I would expect that $zeta(r)$ is irrational for all rationals $r > 1$, but to prove this may be beyond the current state of the art. We don't even know that $zeta(5)$ is irrational.
$endgroup$
– Robert Israel
Aug 19 '18 at 5:54




$begingroup$
I would expect that $zeta(r)$ is irrational for all rationals $r > 1$, but to prove this may be beyond the current state of the art. We don't even know that $zeta(5)$ is irrational.
$endgroup$
– Robert Israel
Aug 19 '18 at 5:54




1




1




$begingroup$
If $zeta$ denotes the Riemann zeta function, please include it in your question.
$endgroup$
– Klangen
Aug 22 '18 at 13:07




$begingroup$
If $zeta$ denotes the Riemann zeta function, please include it in your question.
$endgroup$
– Klangen
Aug 22 '18 at 13:07










3 Answers
3






active

oldest

votes


















17












$begingroup$

I misread $l$ as $1$, but in any case, as a partial result, here's a resolution for the case $l=1$.



Fix $sinmathbbR$, with $s > 1$.



On the interval $(0,infty)$, let
$f(x)=smalldisplaystylefrac1x^larges$.



It's easily verified that
$
displaystyle
int_1^infty !f(x),dx
=
smallfrac1s-1

$.



Consider the infinite series
$
displaystyle
sum_k=1^infty frac1k^s

$.



Since $f$ is positive, continuous, and strictly decreasing, we get
beginalign*
int_1^infty !f(x),dx < ;&sum_k=1^infty frac1k^s < 1+int_1^infty !f(x),dx\[4pt]
implies;smallfrac1s-1 < ;&sum_k=1^infty frac1k^s < 1+smallfrac1s-1\[4pt]
endalign*
If $m$ is a positive integer, then letting $s=1+largefrac1m$, we have $largefrac1s-1=m$, hence
beginalign*
smallfrac1s-1 < ;&sum_k=1^infty frac1k^s < ;1+smallfrac1s-1\[4pt]
implies;m < ;,&zetabigl(1+smallfrac1mbigr) < ;m + 1\[4pt]
endalign*
so $zetabigl(1+largefrac1mbigr)$ is not an integer.






share|cite|improve this answer











$endgroup$








  • 3




    $begingroup$
    Thanks. In fact I have a slightly stronger result can be one of the approach to tackle this problem. Using the Stieltjes series expansion of thte Riemann Zeta function we can show that for $l = 1$, the fractional part of $zeta(1+1/m)$ starts from $pi^2/2 - 1 = 0.6449341$ for $m = 1$ and strictly decreases with $m$ and approaches the limiting value of $1-gamma sim 0.422785$ where $gamma$ is the Euler-Mascheroni constant. Hence $pi^2/2 - 1 le (zeta(1+1/m)) le gamma$. I have been trying to see if this method can be generalized for the case $l > 1$.
    $endgroup$
    – Nilotpal Kanti Sinha
    Aug 19 '18 at 10:52











  • $begingroup$
    Yes, I had that result, but it doesn't go anywhere for $l > 1$.
    $endgroup$
    – quasi
    Aug 19 '18 at 11:02










  • $begingroup$
    Can you resolve the problem for any other value of $l$, other than $l=1$? For example, can you resolve the case $l=2$?
    $endgroup$
    – quasi
    Aug 19 '18 at 11:04










  • $begingroup$
    Note that for positive integers $l,m$, the expression $1+l/m$ can take any rational value greater than $1$, so your question can be recast as asking if there exists a rational number $s > 1$ such that $zeta(s)$ is an integer. My sense (seconding Robert Israel's comment) is that an answer to that question is out of range of current knowledge.
    $endgroup$
    – quasi
    Aug 19 '18 at 11:09











  • $begingroup$
    I think that the case for integer should be easier than the case for deciding rationality. I will outline my approach below since it will be too long for a comment.
    $endgroup$
    – Nilotpal Kanti Sinha
    Aug 20 '18 at 6:36


















15












$begingroup$


Can you resolve the problem for any other value of l, other than l=1?
For example, can you resolve the case l=2?




Yes and in fact I can show that $l ge 2.2*10^4$. Here is the outline of my approach which I am posting as an answer since it is too long to be a comment.



Step 1: The first step was to derive the following result




For every integer $n ge 2$ there exists a positive real $c_n$ such
that $displaystyle zetaBig(1+frac1n-1+c_nBig) = n. $



The first few terms of the asymptotic expansion of $c_n$ in terms of $n$ and the Stieltjes constants $gamma_i$ are



$$ c_n = 1-gamma_0 + fracgamma_1n-1
+ fracgamma_2 + gamma_1 - gamma_0 gamma_1(n-1)^2 + OBig(frac1n^3Big) $$




Step 2: I computed the first few values of $c_n$ but I did not use the above result. Instead I used the following recurrence formula.




Let $alpha_0$ be any positive real and $displaystyle alpha_r+1 = n +
alpha_r - zetaBig(1+frac1n -1 + alpha_rBig); $
then $displaystyle lim_r to inftyalpha_r = c_n$.




Using this we obtained
$$
c_2 approx 0.3724062
$$

$$
c_3 approx 0.3932265
$$

$$
ldots
$$

$$
c_12 approx 0.4164435
$$



Step 3: Show that $l ge 5$



Let $displaystyle zetaBig(1+fraclmBig) in N$ and let $m = lk+d$ where $gcd(l,d) = 1$ and $1 le d < l$.



Clearly, $c_2 le c_n < 1-gamma_0$ or $0.3724062 le c_n < 0.422785$.
Hence we must have $displaystyle 0.3724062 le fracdl < 0.422785$. The fraction with the smallest value of $l$ satisfying this condition is $displaystylefrac25 $ hence $l ge 5$.



Extending the same approach I am able to show that $l > 2.2*10^4$.



Problems with this approach:



With this approach and with powerful computing, we can prove results like if $displaystyle zetaBig(1+fraclmBig) in N $ then $l$ must be greater than some large positive integer but I don't see how this approach will solve the general problem.



Source code:



# Program with maximum n
from time import time
from mpmath import mp

start_time = time()
a = 1
a_end = 10^5
n_max = 2267
# Maximum n is at:', 4468, 1889, 2267

while(a < a_end + 1):
b = 1 + floor(0.372406215900714*a)
while(b <= floor((1 - euler_gamma)*a)):
if(gcd(b,a) == 1):
n = 2
found = 1
while (found == 1):
i = 1
r = 50
c_n = c_n1 = N((1 - euler_gamma), digits = 100)
while (i <= r):
c_n = N(n + c_n - zeta(1 + 1/(n - 1 + c_n)), digits = 10)
c_n1 = N(n + 1 + c_n1 - zeta(1 + 1/(n + 1 - 1 + c_n1)), digits = 10)
i = i + 1
test = N(b/a, digits = 100)
if(c_n < test):
if(test < c_n1):
found = found - 1
# print(b, a, n, c_n, b/a.n(), c_n1)
if (n > n_max):
n_max = n
print("Maximum n is at:", a, b, n_max)
b = b + 1
if(b > floor((1 - euler_gamma)*a)):
found = found - 1
else:
n = n + 1
n = n + 1
else:
n = n + 1
if(found == 1):
found = found - 1
print("Solutions may be found for", a, b/a, c_n, b/a.n(), c_n1)
b = b + 1
else:
b = b + 1
if(a%10^1 == 0):
print("Checked till", a, "Duration", floor(time() - start_time))
a = a + 1





share|cite|improve this answer











$endgroup$








  • 3




    $begingroup$
    Nice work! Seems like a lot of progress. Is this your own problem?
    $endgroup$
    – quasi
    Aug 20 '18 at 9:37






  • 4




    $begingroup$
    Thanks. Yes its my own problem that I worked way back in 2005 and then it got lost among other things. Revisiting it now after 13 years.
    $endgroup$
    – Nilotpal Kanti Sinha
    Aug 20 '18 at 9:52







  • 3




    $begingroup$
    This certainly seems worth publishing. Here is one possibility: tandfonline.com/loi/uexm20
    $endgroup$
    – marty cohen
    Oct 4 '18 at 5:47


















3












$begingroup$

Whilst not a solution, I thought it might be interesting to see What happens if we use the functional equation:



$$zetaleft(1+fracmnright)=2^left(1+fracmnright)pi^fracmnsinleft(fracpi2left(1+fracmnright)right)Gammaleft(-fracmnright)zetaleft(-fracmnright).$$



$zeta(s)$ is known to be rational at the negative integers,



$$zeta(-a)=(-1)^afracB_a+1a+1.$$



You actually get $zeta(-a)=0$ for $a$ even due to the trivial zeros.



But it appears that when you combine this with the functional equation, then in the limit, you get something non-zero and irrational, so that doesn't even give you an integer.



On the other hand if $m/n=2k-1$ is odd then we have



$$zetaleft(2kright)=frac(-1)^k+1B_2k(2pi)^2k2(2k)!,$$



by Euler's formula, which is "almost" rational except for the $pi$ factor. Bugger. So no chance of being integral.



If you modify your problem slightly, then using the above you can produce positive integers, but that's no fun.






share|cite|improve this answer











$endgroup$













    Your Answer








    StackExchange.ready(function()
    var channelOptions =
    tags: "".split(" "),
    id: "69"
    ;
    initTagRenderer("".split(" "), "".split(" "), channelOptions);

    StackExchange.using("externalEditor", function()
    // Have to fire editor after snippets, if snippets enabled
    if (StackExchange.settings.snippets.snippetsEnabled)
    StackExchange.using("snippets", function()
    createEditor();
    );

    else
    createEditor();

    );

    function createEditor()
    StackExchange.prepareEditor(
    heartbeatType: 'answer',
    autoActivateHeartbeat: false,
    convertImagesToLinks: true,
    noModals: true,
    showLowRepImageUploadWarning: true,
    reputationToPostImages: 10,
    bindNavPrevention: true,
    postfix: "",
    imageUploader:
    brandingHtml: "Powered by u003ca class="icon-imgur-white" href="https://imgur.com/"u003eu003c/au003e",
    contentPolicyHtml: "User contributions licensed under u003ca href="https://creativecommons.org/licenses/by-sa/3.0/"u003ecc by-sa 3.0 with attribution requiredu003c/au003e u003ca href="https://stackoverflow.com/legal/content-policy"u003e(content policy)u003c/au003e",
    allowUrls: true
    ,
    noCode: true, onDemand: true,
    discardSelector: ".discard-answer"
    ,immediatelyShowMarkdownHelp:true
    );



    );













    draft saved

    draft discarded


















    StackExchange.ready(
    function ()
    StackExchange.openid.initPostLogin('.new-post-login', 'https%3a%2f%2fmath.stackexchange.com%2fquestions%2f2887379%2fdoes-there-exist-positive-rational-s-for-which-zetas-is-a-positive-intege%23new-answer', 'question_page');

    );

    Post as a guest















    Required, but never shown

























    3 Answers
    3






    active

    oldest

    votes








    3 Answers
    3






    active

    oldest

    votes









    active

    oldest

    votes






    active

    oldest

    votes









    17












    $begingroup$

    I misread $l$ as $1$, but in any case, as a partial result, here's a resolution for the case $l=1$.



    Fix $sinmathbbR$, with $s > 1$.



    On the interval $(0,infty)$, let
    $f(x)=smalldisplaystylefrac1x^larges$.



    It's easily verified that
    $
    displaystyle
    int_1^infty !f(x),dx
    =
    smallfrac1s-1

    $.



    Consider the infinite series
    $
    displaystyle
    sum_k=1^infty frac1k^s

    $.



    Since $f$ is positive, continuous, and strictly decreasing, we get
    beginalign*
    int_1^infty !f(x),dx < ;&sum_k=1^infty frac1k^s < 1+int_1^infty !f(x),dx\[4pt]
    implies;smallfrac1s-1 < ;&sum_k=1^infty frac1k^s < 1+smallfrac1s-1\[4pt]
    endalign*
    If $m$ is a positive integer, then letting $s=1+largefrac1m$, we have $largefrac1s-1=m$, hence
    beginalign*
    smallfrac1s-1 < ;&sum_k=1^infty frac1k^s < ;1+smallfrac1s-1\[4pt]
    implies;m < ;,&zetabigl(1+smallfrac1mbigr) < ;m + 1\[4pt]
    endalign*
    so $zetabigl(1+largefrac1mbigr)$ is not an integer.






    share|cite|improve this answer











    $endgroup$








    • 3




      $begingroup$
      Thanks. In fact I have a slightly stronger result can be one of the approach to tackle this problem. Using the Stieltjes series expansion of thte Riemann Zeta function we can show that for $l = 1$, the fractional part of $zeta(1+1/m)$ starts from $pi^2/2 - 1 = 0.6449341$ for $m = 1$ and strictly decreases with $m$ and approaches the limiting value of $1-gamma sim 0.422785$ where $gamma$ is the Euler-Mascheroni constant. Hence $pi^2/2 - 1 le (zeta(1+1/m)) le gamma$. I have been trying to see if this method can be generalized for the case $l > 1$.
      $endgroup$
      – Nilotpal Kanti Sinha
      Aug 19 '18 at 10:52











    • $begingroup$
      Yes, I had that result, but it doesn't go anywhere for $l > 1$.
      $endgroup$
      – quasi
      Aug 19 '18 at 11:02










    • $begingroup$
      Can you resolve the problem for any other value of $l$, other than $l=1$? For example, can you resolve the case $l=2$?
      $endgroup$
      – quasi
      Aug 19 '18 at 11:04










    • $begingroup$
      Note that for positive integers $l,m$, the expression $1+l/m$ can take any rational value greater than $1$, so your question can be recast as asking if there exists a rational number $s > 1$ such that $zeta(s)$ is an integer. My sense (seconding Robert Israel's comment) is that an answer to that question is out of range of current knowledge.
      $endgroup$
      – quasi
      Aug 19 '18 at 11:09











    • $begingroup$
      I think that the case for integer should be easier than the case for deciding rationality. I will outline my approach below since it will be too long for a comment.
      $endgroup$
      – Nilotpal Kanti Sinha
      Aug 20 '18 at 6:36















    17












    $begingroup$

    I misread $l$ as $1$, but in any case, as a partial result, here's a resolution for the case $l=1$.



    Fix $sinmathbbR$, with $s > 1$.



    On the interval $(0,infty)$, let
    $f(x)=smalldisplaystylefrac1x^larges$.



    It's easily verified that
    $
    displaystyle
    int_1^infty !f(x),dx
    =
    smallfrac1s-1

    $.



    Consider the infinite series
    $
    displaystyle
    sum_k=1^infty frac1k^s

    $.



    Since $f$ is positive, continuous, and strictly decreasing, we get
    beginalign*
    int_1^infty !f(x),dx < ;&sum_k=1^infty frac1k^s < 1+int_1^infty !f(x),dx\[4pt]
    implies;smallfrac1s-1 < ;&sum_k=1^infty frac1k^s < 1+smallfrac1s-1\[4pt]
    endalign*
    If $m$ is a positive integer, then letting $s=1+largefrac1m$, we have $largefrac1s-1=m$, hence
    beginalign*
    smallfrac1s-1 < ;&sum_k=1^infty frac1k^s < ;1+smallfrac1s-1\[4pt]
    implies;m < ;,&zetabigl(1+smallfrac1mbigr) < ;m + 1\[4pt]
    endalign*
    so $zetabigl(1+largefrac1mbigr)$ is not an integer.






    share|cite|improve this answer











    $endgroup$








    • 3




      $begingroup$
      Thanks. In fact I have a slightly stronger result can be one of the approach to tackle this problem. Using the Stieltjes series expansion of thte Riemann Zeta function we can show that for $l = 1$, the fractional part of $zeta(1+1/m)$ starts from $pi^2/2 - 1 = 0.6449341$ for $m = 1$ and strictly decreases with $m$ and approaches the limiting value of $1-gamma sim 0.422785$ where $gamma$ is the Euler-Mascheroni constant. Hence $pi^2/2 - 1 le (zeta(1+1/m)) le gamma$. I have been trying to see if this method can be generalized for the case $l > 1$.
      $endgroup$
      – Nilotpal Kanti Sinha
      Aug 19 '18 at 10:52











    • $begingroup$
      Yes, I had that result, but it doesn't go anywhere for $l > 1$.
      $endgroup$
      – quasi
      Aug 19 '18 at 11:02










    • $begingroup$
      Can you resolve the problem for any other value of $l$, other than $l=1$? For example, can you resolve the case $l=2$?
      $endgroup$
      – quasi
      Aug 19 '18 at 11:04










    • $begingroup$
      Note that for positive integers $l,m$, the expression $1+l/m$ can take any rational value greater than $1$, so your question can be recast as asking if there exists a rational number $s > 1$ such that $zeta(s)$ is an integer. My sense (seconding Robert Israel's comment) is that an answer to that question is out of range of current knowledge.
      $endgroup$
      – quasi
      Aug 19 '18 at 11:09











    • $begingroup$
      I think that the case for integer should be easier than the case for deciding rationality. I will outline my approach below since it will be too long for a comment.
      $endgroup$
      – Nilotpal Kanti Sinha
      Aug 20 '18 at 6:36













    17












    17








    17





    $begingroup$

    I misread $l$ as $1$, but in any case, as a partial result, here's a resolution for the case $l=1$.



    Fix $sinmathbbR$, with $s > 1$.



    On the interval $(0,infty)$, let
    $f(x)=smalldisplaystylefrac1x^larges$.



    It's easily verified that
    $
    displaystyle
    int_1^infty !f(x),dx
    =
    smallfrac1s-1

    $.



    Consider the infinite series
    $
    displaystyle
    sum_k=1^infty frac1k^s

    $.



    Since $f$ is positive, continuous, and strictly decreasing, we get
    beginalign*
    int_1^infty !f(x),dx < ;&sum_k=1^infty frac1k^s < 1+int_1^infty !f(x),dx\[4pt]
    implies;smallfrac1s-1 < ;&sum_k=1^infty frac1k^s < 1+smallfrac1s-1\[4pt]
    endalign*
    If $m$ is a positive integer, then letting $s=1+largefrac1m$, we have $largefrac1s-1=m$, hence
    beginalign*
    smallfrac1s-1 < ;&sum_k=1^infty frac1k^s < ;1+smallfrac1s-1\[4pt]
    implies;m < ;,&zetabigl(1+smallfrac1mbigr) < ;m + 1\[4pt]
    endalign*
    so $zetabigl(1+largefrac1mbigr)$ is not an integer.






    share|cite|improve this answer











    $endgroup$



    I misread $l$ as $1$, but in any case, as a partial result, here's a resolution for the case $l=1$.



    Fix $sinmathbbR$, with $s > 1$.



    On the interval $(0,infty)$, let
    $f(x)=smalldisplaystylefrac1x^larges$.



    It's easily verified that
    $
    displaystyle
    int_1^infty !f(x),dx
    =
    smallfrac1s-1

    $.



    Consider the infinite series
    $
    displaystyle
    sum_k=1^infty frac1k^s

    $.



    Since $f$ is positive, continuous, and strictly decreasing, we get
    beginalign*
    int_1^infty !f(x),dx < ;&sum_k=1^infty frac1k^s < 1+int_1^infty !f(x),dx\[4pt]
    implies;smallfrac1s-1 < ;&sum_k=1^infty frac1k^s < 1+smallfrac1s-1\[4pt]
    endalign*
    If $m$ is a positive integer, then letting $s=1+largefrac1m$, we have $largefrac1s-1=m$, hence
    beginalign*
    smallfrac1s-1 < ;&sum_k=1^infty frac1k^s < ;1+smallfrac1s-1\[4pt]
    implies;m < ;,&zetabigl(1+smallfrac1mbigr) < ;m + 1\[4pt]
    endalign*
    so $zetabigl(1+largefrac1mbigr)$ is not an integer.







    share|cite|improve this answer














    share|cite|improve this answer



    share|cite|improve this answer








    edited Aug 19 '18 at 8:57

























    answered Aug 19 '18 at 8:06









    quasiquasi

    36.2k22664




    36.2k22664







    • 3




      $begingroup$
      Thanks. In fact I have a slightly stronger result can be one of the approach to tackle this problem. Using the Stieltjes series expansion of thte Riemann Zeta function we can show that for $l = 1$, the fractional part of $zeta(1+1/m)$ starts from $pi^2/2 - 1 = 0.6449341$ for $m = 1$ and strictly decreases with $m$ and approaches the limiting value of $1-gamma sim 0.422785$ where $gamma$ is the Euler-Mascheroni constant. Hence $pi^2/2 - 1 le (zeta(1+1/m)) le gamma$. I have been trying to see if this method can be generalized for the case $l > 1$.
      $endgroup$
      – Nilotpal Kanti Sinha
      Aug 19 '18 at 10:52











    • $begingroup$
      Yes, I had that result, but it doesn't go anywhere for $l > 1$.
      $endgroup$
      – quasi
      Aug 19 '18 at 11:02










    • $begingroup$
      Can you resolve the problem for any other value of $l$, other than $l=1$? For example, can you resolve the case $l=2$?
      $endgroup$
      – quasi
      Aug 19 '18 at 11:04










    • $begingroup$
      Note that for positive integers $l,m$, the expression $1+l/m$ can take any rational value greater than $1$, so your question can be recast as asking if there exists a rational number $s > 1$ such that $zeta(s)$ is an integer. My sense (seconding Robert Israel's comment) is that an answer to that question is out of range of current knowledge.
      $endgroup$
      – quasi
      Aug 19 '18 at 11:09











    • $begingroup$
      I think that the case for integer should be easier than the case for deciding rationality. I will outline my approach below since it will be too long for a comment.
      $endgroup$
      – Nilotpal Kanti Sinha
      Aug 20 '18 at 6:36












    • 3




      $begingroup$
      Thanks. In fact I have a slightly stronger result can be one of the approach to tackle this problem. Using the Stieltjes series expansion of thte Riemann Zeta function we can show that for $l = 1$, the fractional part of $zeta(1+1/m)$ starts from $pi^2/2 - 1 = 0.6449341$ for $m = 1$ and strictly decreases with $m$ and approaches the limiting value of $1-gamma sim 0.422785$ where $gamma$ is the Euler-Mascheroni constant. Hence $pi^2/2 - 1 le (zeta(1+1/m)) le gamma$. I have been trying to see if this method can be generalized for the case $l > 1$.
      $endgroup$
      – Nilotpal Kanti Sinha
      Aug 19 '18 at 10:52











    • $begingroup$
      Yes, I had that result, but it doesn't go anywhere for $l > 1$.
      $endgroup$
      – quasi
      Aug 19 '18 at 11:02










    • $begingroup$
      Can you resolve the problem for any other value of $l$, other than $l=1$? For example, can you resolve the case $l=2$?
      $endgroup$
      – quasi
      Aug 19 '18 at 11:04










    • $begingroup$
      Note that for positive integers $l,m$, the expression $1+l/m$ can take any rational value greater than $1$, so your question can be recast as asking if there exists a rational number $s > 1$ such that $zeta(s)$ is an integer. My sense (seconding Robert Israel's comment) is that an answer to that question is out of range of current knowledge.
      $endgroup$
      – quasi
      Aug 19 '18 at 11:09











    • $begingroup$
      I think that the case for integer should be easier than the case for deciding rationality. I will outline my approach below since it will be too long for a comment.
      $endgroup$
      – Nilotpal Kanti Sinha
      Aug 20 '18 at 6:36







    3




    3




    $begingroup$
    Thanks. In fact I have a slightly stronger result can be one of the approach to tackle this problem. Using the Stieltjes series expansion of thte Riemann Zeta function we can show that for $l = 1$, the fractional part of $zeta(1+1/m)$ starts from $pi^2/2 - 1 = 0.6449341$ for $m = 1$ and strictly decreases with $m$ and approaches the limiting value of $1-gamma sim 0.422785$ where $gamma$ is the Euler-Mascheroni constant. Hence $pi^2/2 - 1 le (zeta(1+1/m)) le gamma$. I have been trying to see if this method can be generalized for the case $l > 1$.
    $endgroup$
    – Nilotpal Kanti Sinha
    Aug 19 '18 at 10:52





    $begingroup$
    Thanks. In fact I have a slightly stronger result can be one of the approach to tackle this problem. Using the Stieltjes series expansion of thte Riemann Zeta function we can show that for $l = 1$, the fractional part of $zeta(1+1/m)$ starts from $pi^2/2 - 1 = 0.6449341$ for $m = 1$ and strictly decreases with $m$ and approaches the limiting value of $1-gamma sim 0.422785$ where $gamma$ is the Euler-Mascheroni constant. Hence $pi^2/2 - 1 le (zeta(1+1/m)) le gamma$. I have been trying to see if this method can be generalized for the case $l > 1$.
    $endgroup$
    – Nilotpal Kanti Sinha
    Aug 19 '18 at 10:52













    $begingroup$
    Yes, I had that result, but it doesn't go anywhere for $l > 1$.
    $endgroup$
    – quasi
    Aug 19 '18 at 11:02




    $begingroup$
    Yes, I had that result, but it doesn't go anywhere for $l > 1$.
    $endgroup$
    – quasi
    Aug 19 '18 at 11:02












    $begingroup$
    Can you resolve the problem for any other value of $l$, other than $l=1$? For example, can you resolve the case $l=2$?
    $endgroup$
    – quasi
    Aug 19 '18 at 11:04




    $begingroup$
    Can you resolve the problem for any other value of $l$, other than $l=1$? For example, can you resolve the case $l=2$?
    $endgroup$
    – quasi
    Aug 19 '18 at 11:04












    $begingroup$
    Note that for positive integers $l,m$, the expression $1+l/m$ can take any rational value greater than $1$, so your question can be recast as asking if there exists a rational number $s > 1$ such that $zeta(s)$ is an integer. My sense (seconding Robert Israel's comment) is that an answer to that question is out of range of current knowledge.
    $endgroup$
    – quasi
    Aug 19 '18 at 11:09





    $begingroup$
    Note that for positive integers $l,m$, the expression $1+l/m$ can take any rational value greater than $1$, so your question can be recast as asking if there exists a rational number $s > 1$ such that $zeta(s)$ is an integer. My sense (seconding Robert Israel's comment) is that an answer to that question is out of range of current knowledge.
    $endgroup$
    – quasi
    Aug 19 '18 at 11:09













    $begingroup$
    I think that the case for integer should be easier than the case for deciding rationality. I will outline my approach below since it will be too long for a comment.
    $endgroup$
    – Nilotpal Kanti Sinha
    Aug 20 '18 at 6:36




    $begingroup$
    I think that the case for integer should be easier than the case for deciding rationality. I will outline my approach below since it will be too long for a comment.
    $endgroup$
    – Nilotpal Kanti Sinha
    Aug 20 '18 at 6:36











    15












    $begingroup$


    Can you resolve the problem for any other value of l, other than l=1?
    For example, can you resolve the case l=2?




    Yes and in fact I can show that $l ge 2.2*10^4$. Here is the outline of my approach which I am posting as an answer since it is too long to be a comment.



    Step 1: The first step was to derive the following result




    For every integer $n ge 2$ there exists a positive real $c_n$ such
    that $displaystyle zetaBig(1+frac1n-1+c_nBig) = n. $



    The first few terms of the asymptotic expansion of $c_n$ in terms of $n$ and the Stieltjes constants $gamma_i$ are



    $$ c_n = 1-gamma_0 + fracgamma_1n-1
    + fracgamma_2 + gamma_1 - gamma_0 gamma_1(n-1)^2 + OBig(frac1n^3Big) $$




    Step 2: I computed the first few values of $c_n$ but I did not use the above result. Instead I used the following recurrence formula.




    Let $alpha_0$ be any positive real and $displaystyle alpha_r+1 = n +
    alpha_r - zetaBig(1+frac1n -1 + alpha_rBig); $
    then $displaystyle lim_r to inftyalpha_r = c_n$.




    Using this we obtained
    $$
    c_2 approx 0.3724062
    $$

    $$
    c_3 approx 0.3932265
    $$

    $$
    ldots
    $$

    $$
    c_12 approx 0.4164435
    $$



    Step 3: Show that $l ge 5$



    Let $displaystyle zetaBig(1+fraclmBig) in N$ and let $m = lk+d$ where $gcd(l,d) = 1$ and $1 le d < l$.



    Clearly, $c_2 le c_n < 1-gamma_0$ or $0.3724062 le c_n < 0.422785$.
    Hence we must have $displaystyle 0.3724062 le fracdl < 0.422785$. The fraction with the smallest value of $l$ satisfying this condition is $displaystylefrac25 $ hence $l ge 5$.



    Extending the same approach I am able to show that $l > 2.2*10^4$.



    Problems with this approach:



    With this approach and with powerful computing, we can prove results like if $displaystyle zetaBig(1+fraclmBig) in N $ then $l$ must be greater than some large positive integer but I don't see how this approach will solve the general problem.



    Source code:



    # Program with maximum n
    from time import time
    from mpmath import mp

    start_time = time()
    a = 1
    a_end = 10^5
    n_max = 2267
    # Maximum n is at:', 4468, 1889, 2267

    while(a < a_end + 1):
    b = 1 + floor(0.372406215900714*a)
    while(b <= floor((1 - euler_gamma)*a)):
    if(gcd(b,a) == 1):
    n = 2
    found = 1
    while (found == 1):
    i = 1
    r = 50
    c_n = c_n1 = N((1 - euler_gamma), digits = 100)
    while (i <= r):
    c_n = N(n + c_n - zeta(1 + 1/(n - 1 + c_n)), digits = 10)
    c_n1 = N(n + 1 + c_n1 - zeta(1 + 1/(n + 1 - 1 + c_n1)), digits = 10)
    i = i + 1
    test = N(b/a, digits = 100)
    if(c_n < test):
    if(test < c_n1):
    found = found - 1
    # print(b, a, n, c_n, b/a.n(), c_n1)
    if (n > n_max):
    n_max = n
    print("Maximum n is at:", a, b, n_max)
    b = b + 1
    if(b > floor((1 - euler_gamma)*a)):
    found = found - 1
    else:
    n = n + 1
    n = n + 1
    else:
    n = n + 1
    if(found == 1):
    found = found - 1
    print("Solutions may be found for", a, b/a, c_n, b/a.n(), c_n1)
    b = b + 1
    else:
    b = b + 1
    if(a%10^1 == 0):
    print("Checked till", a, "Duration", floor(time() - start_time))
    a = a + 1





    share|cite|improve this answer











    $endgroup$








    • 3




      $begingroup$
      Nice work! Seems like a lot of progress. Is this your own problem?
      $endgroup$
      – quasi
      Aug 20 '18 at 9:37






    • 4




      $begingroup$
      Thanks. Yes its my own problem that I worked way back in 2005 and then it got lost among other things. Revisiting it now after 13 years.
      $endgroup$
      – Nilotpal Kanti Sinha
      Aug 20 '18 at 9:52







    • 3




      $begingroup$
      This certainly seems worth publishing. Here is one possibility: tandfonline.com/loi/uexm20
      $endgroup$
      – marty cohen
      Oct 4 '18 at 5:47















    15












    $begingroup$


    Can you resolve the problem for any other value of l, other than l=1?
    For example, can you resolve the case l=2?




    Yes and in fact I can show that $l ge 2.2*10^4$. Here is the outline of my approach which I am posting as an answer since it is too long to be a comment.



    Step 1: The first step was to derive the following result




    For every integer $n ge 2$ there exists a positive real $c_n$ such
    that $displaystyle zetaBig(1+frac1n-1+c_nBig) = n. $



    The first few terms of the asymptotic expansion of $c_n$ in terms of $n$ and the Stieltjes constants $gamma_i$ are



    $$ c_n = 1-gamma_0 + fracgamma_1n-1
    + fracgamma_2 + gamma_1 - gamma_0 gamma_1(n-1)^2 + OBig(frac1n^3Big) $$




    Step 2: I computed the first few values of $c_n$ but I did not use the above result. Instead I used the following recurrence formula.




    Let $alpha_0$ be any positive real and $displaystyle alpha_r+1 = n +
    alpha_r - zetaBig(1+frac1n -1 + alpha_rBig); $
    then $displaystyle lim_r to inftyalpha_r = c_n$.




    Using this we obtained
    $$
    c_2 approx 0.3724062
    $$

    $$
    c_3 approx 0.3932265
    $$

    $$
    ldots
    $$

    $$
    c_12 approx 0.4164435
    $$



    Step 3: Show that $l ge 5$



    Let $displaystyle zetaBig(1+fraclmBig) in N$ and let $m = lk+d$ where $gcd(l,d) = 1$ and $1 le d < l$.



    Clearly, $c_2 le c_n < 1-gamma_0$ or $0.3724062 le c_n < 0.422785$.
    Hence we must have $displaystyle 0.3724062 le fracdl < 0.422785$. The fraction with the smallest value of $l$ satisfying this condition is $displaystylefrac25 $ hence $l ge 5$.



    Extending the same approach I am able to show that $l > 2.2*10^4$.



    Problems with this approach:



    With this approach and with powerful computing, we can prove results like if $displaystyle zetaBig(1+fraclmBig) in N $ then $l$ must be greater than some large positive integer but I don't see how this approach will solve the general problem.



    Source code:



    # Program with maximum n
    from time import time
    from mpmath import mp

    start_time = time()
    a = 1
    a_end = 10^5
    n_max = 2267
    # Maximum n is at:', 4468, 1889, 2267

    while(a < a_end + 1):
    b = 1 + floor(0.372406215900714*a)
    while(b <= floor((1 - euler_gamma)*a)):
    if(gcd(b,a) == 1):
    n = 2
    found = 1
    while (found == 1):
    i = 1
    r = 50
    c_n = c_n1 = N((1 - euler_gamma), digits = 100)
    while (i <= r):
    c_n = N(n + c_n - zeta(1 + 1/(n - 1 + c_n)), digits = 10)
    c_n1 = N(n + 1 + c_n1 - zeta(1 + 1/(n + 1 - 1 + c_n1)), digits = 10)
    i = i + 1
    test = N(b/a, digits = 100)
    if(c_n < test):
    if(test < c_n1):
    found = found - 1
    # print(b, a, n, c_n, b/a.n(), c_n1)
    if (n > n_max):
    n_max = n
    print("Maximum n is at:", a, b, n_max)
    b = b + 1
    if(b > floor((1 - euler_gamma)*a)):
    found = found - 1
    else:
    n = n + 1
    n = n + 1
    else:
    n = n + 1
    if(found == 1):
    found = found - 1
    print("Solutions may be found for", a, b/a, c_n, b/a.n(), c_n1)
    b = b + 1
    else:
    b = b + 1
    if(a%10^1 == 0):
    print("Checked till", a, "Duration", floor(time() - start_time))
    a = a + 1





    share|cite|improve this answer











    $endgroup$








    • 3




      $begingroup$
      Nice work! Seems like a lot of progress. Is this your own problem?
      $endgroup$
      – quasi
      Aug 20 '18 at 9:37






    • 4




      $begingroup$
      Thanks. Yes its my own problem that I worked way back in 2005 and then it got lost among other things. Revisiting it now after 13 years.
      $endgroup$
      – Nilotpal Kanti Sinha
      Aug 20 '18 at 9:52







    • 3




      $begingroup$
      This certainly seems worth publishing. Here is one possibility: tandfonline.com/loi/uexm20
      $endgroup$
      – marty cohen
      Oct 4 '18 at 5:47













    15












    15








    15





    $begingroup$


    Can you resolve the problem for any other value of l, other than l=1?
    For example, can you resolve the case l=2?




    Yes and in fact I can show that $l ge 2.2*10^4$. Here is the outline of my approach which I am posting as an answer since it is too long to be a comment.



    Step 1: The first step was to derive the following result




    For every integer $n ge 2$ there exists a positive real $c_n$ such
    that $displaystyle zetaBig(1+frac1n-1+c_nBig) = n. $



    The first few terms of the asymptotic expansion of $c_n$ in terms of $n$ and the Stieltjes constants $gamma_i$ are



    $$ c_n = 1-gamma_0 + fracgamma_1n-1
    + fracgamma_2 + gamma_1 - gamma_0 gamma_1(n-1)^2 + OBig(frac1n^3Big) $$




    Step 2: I computed the first few values of $c_n$ but I did not use the above result. Instead I used the following recurrence formula.




    Let $alpha_0$ be any positive real and $displaystyle alpha_r+1 = n +
    alpha_r - zetaBig(1+frac1n -1 + alpha_rBig); $
    then $displaystyle lim_r to inftyalpha_r = c_n$.




    Using this we obtained
    $$
    c_2 approx 0.3724062
    $$

    $$
    c_3 approx 0.3932265
    $$

    $$
    ldots
    $$

    $$
    c_12 approx 0.4164435
    $$



    Step 3: Show that $l ge 5$



    Let $displaystyle zetaBig(1+fraclmBig) in N$ and let $m = lk+d$ where $gcd(l,d) = 1$ and $1 le d < l$.



    Clearly, $c_2 le c_n < 1-gamma_0$ or $0.3724062 le c_n < 0.422785$.
    Hence we must have $displaystyle 0.3724062 le fracdl < 0.422785$. The fraction with the smallest value of $l$ satisfying this condition is $displaystylefrac25 $ hence $l ge 5$.



    Extending the same approach I am able to show that $l > 2.2*10^4$.



    Problems with this approach:



    With this approach and with powerful computing, we can prove results like if $displaystyle zetaBig(1+fraclmBig) in N $ then $l$ must be greater than some large positive integer but I don't see how this approach will solve the general problem.



    Source code:



    # Program with maximum n
    from time import time
    from mpmath import mp

    start_time = time()
    a = 1
    a_end = 10^5
    n_max = 2267
    # Maximum n is at:', 4468, 1889, 2267

    while(a < a_end + 1):
    b = 1 + floor(0.372406215900714*a)
    while(b <= floor((1 - euler_gamma)*a)):
    if(gcd(b,a) == 1):
    n = 2
    found = 1
    while (found == 1):
    i = 1
    r = 50
    c_n = c_n1 = N((1 - euler_gamma), digits = 100)
    while (i <= r):
    c_n = N(n + c_n - zeta(1 + 1/(n - 1 + c_n)), digits = 10)
    c_n1 = N(n + 1 + c_n1 - zeta(1 + 1/(n + 1 - 1 + c_n1)), digits = 10)
    i = i + 1
    test = N(b/a, digits = 100)
    if(c_n < test):
    if(test < c_n1):
    found = found - 1
    # print(b, a, n, c_n, b/a.n(), c_n1)
    if (n > n_max):
    n_max = n
    print("Maximum n is at:", a, b, n_max)
    b = b + 1
    if(b > floor((1 - euler_gamma)*a)):
    found = found - 1
    else:
    n = n + 1
    n = n + 1
    else:
    n = n + 1
    if(found == 1):
    found = found - 1
    print("Solutions may be found for", a, b/a, c_n, b/a.n(), c_n1)
    b = b + 1
    else:
    b = b + 1
    if(a%10^1 == 0):
    print("Checked till", a, "Duration", floor(time() - start_time))
    a = a + 1





    share|cite|improve this answer











    $endgroup$




    Can you resolve the problem for any other value of l, other than l=1?
    For example, can you resolve the case l=2?




    Yes and in fact I can show that $l ge 2.2*10^4$. Here is the outline of my approach which I am posting as an answer since it is too long to be a comment.



    Step 1: The first step was to derive the following result




    For every integer $n ge 2$ there exists a positive real $c_n$ such
    that $displaystyle zetaBig(1+frac1n-1+c_nBig) = n. $



    The first few terms of the asymptotic expansion of $c_n$ in terms of $n$ and the Stieltjes constants $gamma_i$ are



    $$ c_n = 1-gamma_0 + fracgamma_1n-1
    + fracgamma_2 + gamma_1 - gamma_0 gamma_1(n-1)^2 + OBig(frac1n^3Big) $$




    Step 2: I computed the first few values of $c_n$ but I did not use the above result. Instead I used the following recurrence formula.




    Let $alpha_0$ be any positive real and $displaystyle alpha_r+1 = n +
    alpha_r - zetaBig(1+frac1n -1 + alpha_rBig); $
    then $displaystyle lim_r to inftyalpha_r = c_n$.




    Using this we obtained
    $$
    c_2 approx 0.3724062
    $$

    $$
    c_3 approx 0.3932265
    $$

    $$
    ldots
    $$

    $$
    c_12 approx 0.4164435
    $$



    Step 3: Show that $l ge 5$



    Let $displaystyle zetaBig(1+fraclmBig) in N$ and let $m = lk+d$ where $gcd(l,d) = 1$ and $1 le d < l$.



    Clearly, $c_2 le c_n < 1-gamma_0$ or $0.3724062 le c_n < 0.422785$.
    Hence we must have $displaystyle 0.3724062 le fracdl < 0.422785$. The fraction with the smallest value of $l$ satisfying this condition is $displaystylefrac25 $ hence $l ge 5$.



    Extending the same approach I am able to show that $l > 2.2*10^4$.



    Problems with this approach:



    With this approach and with powerful computing, we can prove results like if $displaystyle zetaBig(1+fraclmBig) in N $ then $l$ must be greater than some large positive integer but I don't see how this approach will solve the general problem.



    Source code:



    # Program with maximum n
    from time import time
    from mpmath import mp

    start_time = time()
    a = 1
    a_end = 10^5
    n_max = 2267
    # Maximum n is at:', 4468, 1889, 2267

    while(a < a_end + 1):
    b = 1 + floor(0.372406215900714*a)
    while(b <= floor((1 - euler_gamma)*a)):
    if(gcd(b,a) == 1):
    n = 2
    found = 1
    while (found == 1):
    i = 1
    r = 50
    c_n = c_n1 = N((1 - euler_gamma), digits = 100)
    while (i <= r):
    c_n = N(n + c_n - zeta(1 + 1/(n - 1 + c_n)), digits = 10)
    c_n1 = N(n + 1 + c_n1 - zeta(1 + 1/(n + 1 - 1 + c_n1)), digits = 10)
    i = i + 1
    test = N(b/a, digits = 100)
    if(c_n < test):
    if(test < c_n1):
    found = found - 1
    # print(b, a, n, c_n, b/a.n(), c_n1)
    if (n > n_max):
    n_max = n
    print("Maximum n is at:", a, b, n_max)
    b = b + 1
    if(b > floor((1 - euler_gamma)*a)):
    found = found - 1
    else:
    n = n + 1
    n = n + 1
    else:
    n = n + 1
    if(found == 1):
    found = found - 1
    print("Solutions may be found for", a, b/a, c_n, b/a.n(), c_n1)
    b = b + 1
    else:
    b = b + 1
    if(a%10^1 == 0):
    print("Checked till", a, "Duration", floor(time() - start_time))
    a = a + 1






    share|cite|improve this answer














    share|cite|improve this answer



    share|cite|improve this answer








    edited Jan 9 at 5:24

























    answered Aug 20 '18 at 7:57









    Nilotpal Kanti SinhaNilotpal Kanti Sinha

    4,67121641




    4,67121641







    • 3




      $begingroup$
      Nice work! Seems like a lot of progress. Is this your own problem?
      $endgroup$
      – quasi
      Aug 20 '18 at 9:37






    • 4




      $begingroup$
      Thanks. Yes its my own problem that I worked way back in 2005 and then it got lost among other things. Revisiting it now after 13 years.
      $endgroup$
      – Nilotpal Kanti Sinha
      Aug 20 '18 at 9:52







    • 3




      $begingroup$
      This certainly seems worth publishing. Here is one possibility: tandfonline.com/loi/uexm20
      $endgroup$
      – marty cohen
      Oct 4 '18 at 5:47












    • 3




      $begingroup$
      Nice work! Seems like a lot of progress. Is this your own problem?
      $endgroup$
      – quasi
      Aug 20 '18 at 9:37






    • 4




      $begingroup$
      Thanks. Yes its my own problem that I worked way back in 2005 and then it got lost among other things. Revisiting it now after 13 years.
      $endgroup$
      – Nilotpal Kanti Sinha
      Aug 20 '18 at 9:52







    • 3




      $begingroup$
      This certainly seems worth publishing. Here is one possibility: tandfonline.com/loi/uexm20
      $endgroup$
      – marty cohen
      Oct 4 '18 at 5:47







    3




    3




    $begingroup$
    Nice work! Seems like a lot of progress. Is this your own problem?
    $endgroup$
    – quasi
    Aug 20 '18 at 9:37




    $begingroup$
    Nice work! Seems like a lot of progress. Is this your own problem?
    $endgroup$
    – quasi
    Aug 20 '18 at 9:37




    4




    4




    $begingroup$
    Thanks. Yes its my own problem that I worked way back in 2005 and then it got lost among other things. Revisiting it now after 13 years.
    $endgroup$
    – Nilotpal Kanti Sinha
    Aug 20 '18 at 9:52





    $begingroup$
    Thanks. Yes its my own problem that I worked way back in 2005 and then it got lost among other things. Revisiting it now after 13 years.
    $endgroup$
    – Nilotpal Kanti Sinha
    Aug 20 '18 at 9:52





    3




    3




    $begingroup$
    This certainly seems worth publishing. Here is one possibility: tandfonline.com/loi/uexm20
    $endgroup$
    – marty cohen
    Oct 4 '18 at 5:47




    $begingroup$
    This certainly seems worth publishing. Here is one possibility: tandfonline.com/loi/uexm20
    $endgroup$
    – marty cohen
    Oct 4 '18 at 5:47











    3












    $begingroup$

    Whilst not a solution, I thought it might be interesting to see What happens if we use the functional equation:



    $$zetaleft(1+fracmnright)=2^left(1+fracmnright)pi^fracmnsinleft(fracpi2left(1+fracmnright)right)Gammaleft(-fracmnright)zetaleft(-fracmnright).$$



    $zeta(s)$ is known to be rational at the negative integers,



    $$zeta(-a)=(-1)^afracB_a+1a+1.$$



    You actually get $zeta(-a)=0$ for $a$ even due to the trivial zeros.



    But it appears that when you combine this with the functional equation, then in the limit, you get something non-zero and irrational, so that doesn't even give you an integer.



    On the other hand if $m/n=2k-1$ is odd then we have



    $$zetaleft(2kright)=frac(-1)^k+1B_2k(2pi)^2k2(2k)!,$$



    by Euler's formula, which is "almost" rational except for the $pi$ factor. Bugger. So no chance of being integral.



    If you modify your problem slightly, then using the above you can produce positive integers, but that's no fun.






    share|cite|improve this answer











    $endgroup$

















      3












      $begingroup$

      Whilst not a solution, I thought it might be interesting to see What happens if we use the functional equation:



      $$zetaleft(1+fracmnright)=2^left(1+fracmnright)pi^fracmnsinleft(fracpi2left(1+fracmnright)right)Gammaleft(-fracmnright)zetaleft(-fracmnright).$$



      $zeta(s)$ is known to be rational at the negative integers,



      $$zeta(-a)=(-1)^afracB_a+1a+1.$$



      You actually get $zeta(-a)=0$ for $a$ even due to the trivial zeros.



      But it appears that when you combine this with the functional equation, then in the limit, you get something non-zero and irrational, so that doesn't even give you an integer.



      On the other hand if $m/n=2k-1$ is odd then we have



      $$zetaleft(2kright)=frac(-1)^k+1B_2k(2pi)^2k2(2k)!,$$



      by Euler's formula, which is "almost" rational except for the $pi$ factor. Bugger. So no chance of being integral.



      If you modify your problem slightly, then using the above you can produce positive integers, but that's no fun.






      share|cite|improve this answer











      $endgroup$















        3












        3








        3





        $begingroup$

        Whilst not a solution, I thought it might be interesting to see What happens if we use the functional equation:



        $$zetaleft(1+fracmnright)=2^left(1+fracmnright)pi^fracmnsinleft(fracpi2left(1+fracmnright)right)Gammaleft(-fracmnright)zetaleft(-fracmnright).$$



        $zeta(s)$ is known to be rational at the negative integers,



        $$zeta(-a)=(-1)^afracB_a+1a+1.$$



        You actually get $zeta(-a)=0$ for $a$ even due to the trivial zeros.



        But it appears that when you combine this with the functional equation, then in the limit, you get something non-zero and irrational, so that doesn't even give you an integer.



        On the other hand if $m/n=2k-1$ is odd then we have



        $$zetaleft(2kright)=frac(-1)^k+1B_2k(2pi)^2k2(2k)!,$$



        by Euler's formula, which is "almost" rational except for the $pi$ factor. Bugger. So no chance of being integral.



        If you modify your problem slightly, then using the above you can produce positive integers, but that's no fun.






        share|cite|improve this answer











        $endgroup$



        Whilst not a solution, I thought it might be interesting to see What happens if we use the functional equation:



        $$zetaleft(1+fracmnright)=2^left(1+fracmnright)pi^fracmnsinleft(fracpi2left(1+fracmnright)right)Gammaleft(-fracmnright)zetaleft(-fracmnright).$$



        $zeta(s)$ is known to be rational at the negative integers,



        $$zeta(-a)=(-1)^afracB_a+1a+1.$$



        You actually get $zeta(-a)=0$ for $a$ even due to the trivial zeros.



        But it appears that when you combine this with the functional equation, then in the limit, you get something non-zero and irrational, so that doesn't even give you an integer.



        On the other hand if $m/n=2k-1$ is odd then we have



        $$zetaleft(2kright)=frac(-1)^k+1B_2k(2pi)^2k2(2k)!,$$



        by Euler's formula, which is "almost" rational except for the $pi$ factor. Bugger. So no chance of being integral.



        If you modify your problem slightly, then using the above you can produce positive integers, but that's no fun.







        share|cite|improve this answer














        share|cite|improve this answer



        share|cite|improve this answer








        edited Feb 25 at 8:07

























        answered Feb 25 at 8:01









        AntinousAntinous

        5,84842453




        5,84842453



























            draft saved

            draft discarded
















































            Thanks for contributing an answer to Mathematics Stack Exchange!


            • Please be sure to answer the question. Provide details and share your research!

            But avoid


            • Asking for help, clarification, or responding to other answers.

            • Making statements based on opinion; back them up with references or personal experience.

            Use MathJax to format equations. MathJax reference.


            To learn more, see our tips on writing great answers.




            draft saved


            draft discarded














            StackExchange.ready(
            function ()
            StackExchange.openid.initPostLogin('.new-post-login', 'https%3a%2f%2fmath.stackexchange.com%2fquestions%2f2887379%2fdoes-there-exist-positive-rational-s-for-which-zetas-is-a-positive-intege%23new-answer', 'question_page');

            );

            Post as a guest















            Required, but never shown





















































            Required, but never shown














            Required, but never shown












            Required, but never shown







            Required, but never shown

































            Required, but never shown














            Required, but never shown












            Required, but never shown







            Required, but never shown







            Popular posts from this blog

            Lowndes Grove History Architecture References Navigation menu32°48′6″N 79°57′58″W / 32.80167°N 79.96611°W / 32.80167; -79.9661132°48′6″N 79°57′58″W / 32.80167°N 79.96611°W / 32.80167; -79.9661178002500"National Register Information System"Historic houses of South Carolina"Lowndes Grove""+32° 48' 6.00", −79° 57' 58.00""Lowndes Grove, Charleston County (260 St. Margaret St., Charleston)""Lowndes Grove"The Charleston ExpositionIt Happened in South Carolina"Lowndes Grove (House), Saint Margaret Street & Sixth Avenue, Charleston, Charleston County, SC(Photographs)"Plantations of the Carolina Low Countrye

            random experiment with two different functions on unit interval Announcing the arrival of Valued Associate #679: Cesar Manara Planned maintenance scheduled April 23, 2019 at 00:00UTC (8:00pm US/Eastern)Random variable and probability space notionsRandom Walk with EdgesFinding functions where the increase over a random interval is Poisson distributedNumber of days until dayCan an observed event in fact be of zero probability?Unit random processmodels of coins and uniform distributionHow to get the number of successes given $n$ trials , probability $P$ and a random variable $X$Absorbing Markov chain in a computer. Is “almost every” turned into always convergence in computer executions?Stopped random walk is not uniformly integrable

            How should I support this large drywall patch? Planned maintenance scheduled April 23, 2019 at 00:00UTC (8:00pm US/Eastern) Announcing the arrival of Valued Associate #679: Cesar Manara Unicorn Meta Zoo #1: Why another podcast?How do I cover large gaps in drywall?How do I keep drywall around a patch from crumbling?Can I glue a second layer of drywall?How to patch long strip on drywall?Large drywall patch: how to avoid bulging seams?Drywall Mesh Patch vs. Bulge? To remove or not to remove?How to fix this drywall job?Prep drywall before backsplashWhat's the best way to fix this horrible drywall patch job?Drywall patching using 3M Patch Plus Primer